Segunda derivada de la expresión delta de Dirac

me he encontrado con la expresión

F ( X ) d ( X a ) d ( X a ) d X
donde la prima representa la derivada.

Por lo general, con los derivados de la distribución delta de Dirac, integraría parcialmente, pero aquí sigo corriendo en círculos. Lo que probé es

F d d = ( F d ) d = F d d F d d = F d d + ( F d ) d = F d d + F d d + F d d = F d d
o si tomo el otro término para la segunda integración parcial
F d d = ( F d ) d = ( F d ) d = F d d + 2 F d d + F d d 0 = F d d + 2 F d d
que también podría haber obtenido de la integración parcial elemental de F d 2 .

¿Qué otras opciones tengo? Los cuadrados de las funciones delta, etc. no son un problema.

Simplemente aplique la regla regular para integrar una función delta, sustituya en cualquier otro lugar X = a . Tus rendimientos integrales F ( a ) d ( 0 ) .
@JamalS Pero los valores de una derivada de la distribución delta no están definidos. Solo el d sin ningún derivado puede ser evaluado.
¿Puede dar el contexto específico en el que surge esta integral?
La función delta doblemente derivada surge en teorías con dimensiones más altas, cuando calcula los términos FI inducidos por bucle. Si acopla este término FI a un escalar de brana y no desea compensar el término FI por otros medios (como flujos de fondo), en la acción aparece una combinación como la descrita. Para derivar la acción 4D efectiva, necesito integrar las dimensiones internas, dándome la integral anterior.
No conozco ninguna definición razonable del cuadrado de una función delta. ¿Cómo lo defines?
No es una definición tan razonable, pero los infinitos que surgen se pueden reescribir de tal manera que cancelen partes divergentes de términos de orden superior. d 2 todavía no está muy bien definido, pero puedo deshacerme del d ( 0 ) eso sigue y para la mayoría de los físicos esto es suficiente; en realidad, esa es la razón por la que puse esto aquí y no en math.stackexchange.com
Con respecto al producto de las distribuciones delta de Dirac: mathoverflow.net/q/48067 , physics.stackexchange.com/q/47934/2451 y los enlaces incluidos.
Solo para que conste, matemáticamente las derivadas de las funciones delta (o cualquier otra distribución) están bien definidas (como distribuciones): d ( X ) es la distribución tal que d ( X ) F ( X ) d X = F ( 0 ) .
@yuggib: Sí, creo que en general F ( X ) d ( norte ) ( X a ) d X = ( 1 ) norte F ( norte ) ( a ) .
Entiendo que la integral es de -∞ a +∞. Entonces, es útil hacer el cambio de variable u = x - a . Entonces, tendrás ∫f(u+a)δ(u)δ′′(u)du . Por lo demás, no veo un posible escape de δ′′, también lo intenté. Entonces, integremos directamente tu integrando inicial. Obtienes, ∫f(u+a)δ(u)δ′′(u)du = f(a)δ′′(0). Desafortunadamente, δ′′(0) = −∞ .
Comentario a la pregunta (v2): Haciéndose eco del comentario de @JamalS: Considere agregar referencias para recibir respuestas útiles y enfocadas.
Creo que puedes multiplicar distribuciones solo cuando las singularidades de cada una de las distribuciones no están una encima de la otra. El hecho de que tenga dos distribuciones con singularidades en el mismo punto hace que parezca que está mal definido.

Respuestas (1)

Solo estaba jugando con las cosas y obtuve esto, avíseme si le parece una idea útil (o si me falta algo que hace que esto sea inútil).

Denotar d k ( X ) = 1 k π mi ( X / k ) 2 . Obtienes el conocido resultado (completamente matemáticamente riguroso):

límite k 0 F ( X ) d k ( X ) d X = F ( 0 ) por portarse lo suficientemente bien F .

Así que echemos un vistazo a:

límite k 0 ( C 0 + C 1 X + C 2 X 2 + ) d k ( X ) d k ( X ) d X = F ( 0 )

Usando la conocida técnica de Demostración por Mathematica:

delta[x_] := 1/(k Sqrt[ Pi]) E^(-(x/k)^2);
g[x_] = FullSimplify[delta''[x] delta[x] x^n];
Integrate[g[x], {x, -Infinity, Infinity}, Assumptions -> {Element[n, Integers], a > 0}]

obtengo el resultado:

X norte d k ( X ) d k ( X ) d X = 2 norte 2 3 2 ( ( 1 ) norte + 1 ) ( norte 1 ) k norte 3 Γ ( norte + 1 2 ) π

Como k 0 + , el valor es cero para norte = 1 , 3 , 4 , 5 , 6 , 7 , y diverge por norte = 0 , 2 , como 1 2 π k 3 Para el norte = 0 caso y como 1 4 k 2 π Para el norte = 2 caso.

Buena idea para mirar F ( X ) = X norte ! de hecho para norte > 2 tenemos X norte d = 0 , por lo que la integral tiene sentido y es 0. Además tenemos X 0 d = d , X 1 d = 2 d y X 2 d = d . Tenga en cuenta que su resultado de que obtenemos 0 para norte = 1 , así como el orden de crecimiento, son artefactos de la secuencia que eligió para converger a d .
@doetoe ¿Cómo puedes estar seguro de que son solo artefactos? (aunque creo que tienes razón, ya que el 2 π parece demasiado específico). El proceso funciona exactamente (sin artefactos) para d ( norte ) ( X )
@doetoe en realidad, borra eso, supongo que es un poco obvio que son artefactos, y solo el k = 0 el comportamiento (que no existe) es la parte importante.